- PowerScore Staff
- Posts: 5972
- Joined: Mar 25, 2011
- Tue Jul 19, 2016 4:16 pm
#27320
Complete Question Explanation
The correct answer choice is (D)
Answer choice (A) is incorrect because if the main-dish recipe has three flavorings, then the appetizer recipe will have four flavorings, and from the first rule the appetizer recipe can only have at most three flavorings.
Answer choice (B) is incorrect because F and N cannot be included in the same recipe per the second rule.
Answer choice (C) is incorrect because from the third rule one of S or T must be included in the main-dish recipe.
Answer choice (D) is the correct answer choice.
Answer choice (E) is incorrect because S and T cannot be included in the same recipe.
The correct answer choice is (D)
Answer choice (A) is incorrect because if the main-dish recipe has three flavorings, then the appetizer recipe will have four flavorings, and from the first rule the appetizer recipe can only have at most three flavorings.
Answer choice (B) is incorrect because F and N cannot be included in the same recipe per the second rule.
Answer choice (C) is incorrect because from the third rule one of S or T must be included in the main-dish recipe.
Answer choice (D) is the correct answer choice.
Answer choice (E) is incorrect because S and T cannot be included in the same recipe.
Dave Killoran
PowerScore Test Preparation
Follow me on X/Twitter at http://twitter.com/DaveKilloran
My LSAT Articles: http://blog.powerscore.com/lsat/author/dave-killoran
PowerScore Podcast: http://www.powerscore.com/lsat/podcast/
PowerScore Test Preparation
Follow me on X/Twitter at http://twitter.com/DaveKilloran
My LSAT Articles: http://blog.powerscore.com/lsat/author/dave-killoran
PowerScore Podcast: http://www.powerscore.com/lsat/podcast/